Praxis Core Reading Practice Test

The Praxis 5713 Core Reading test will measure your ability to read, analyze, and comprehend multiple documents. These will include a wide variety of text passages as well as graphic-based documents such as charts, graphs, and indexes. Use our free Praxis Core Reading practice test as a part of your test prep. This online test includes challenging practice questions along with detailed answer explanations.

Congratulations - you have completed . You scored %%SCORE%% out of %%TOTAL%%. Your performance has been rated as %%RATING%%
Your answers are highlighted below.
Question 1

Read the passage below; then answer questions 1–5.

Jonathan Swift’s dark view of humanity is expressed in his novel, Gulliver’s Travels. Swift believed that society, which is meant to protect people from inequality and injustice, actually encourages the worst evils, promotes immorality, and allows injustices to occur. In Gulliver’s Travels, Swift asks whether “civilization” is only an elaborate cover for the lowest forms of human behavior. Swift raises this question through satire, a literary form which he brings to perfection in Gulliver’s Travels. Following in the great tradition of classical satirists, Swift uses Gulliver’s Travels to point out the underlying ridiculousness of the society around him. In one section of Gulliver’s Travels, for example, Gulliver describes his native England to the king of a land of giants. Gulliver talks enthusiastically to the giant king about the class system, laws, constitution, military glory, and history of his homeland. After carefully listening to Gulliver’s speech, however, the king proceeds to point out the many shortcomings of Gulliver’s native land. Gulliver is unable to come up with adequate responses to the criticism. He can only stand there in embarrassed silence as the king eventually concludes that the institutions of the human world should be condemned.
 

The passage is primarily concerned with

A
Jonathan’s Swift’s command of written English.
B
The popularity of Jonathan Swift’s Gulliver’s Travels.
C
The public response to Jonathan Swift’s Gulliver’s Travels.
D
Jonathan Swift’s criticism of humanity and civilization.
E
The importance of satire like Jonathan Swift’s Gulliver’s Travels in the modern world.
Question 1 Explanation: 
Answer choice (D) is correct because the passage focuses on Swifts critique of society, not on the book itself. The passage doesn’t directly discuss Swift’s command of written English (A), nor does it remark on the popularity of Swift’s novel (B) or how people responded to it (C). It might be tempting to pick answer choice (E) because the reader can get a feeling that Swift’s novel was important from the passage, but this is not the focus of the passage.
Question 2
Jonathan Swift’s dark view of humanity is expressed in his novel, Gulliver’s Travels. Swift believed that society, which is meant to protect people from inequality and injustice, actually encourages the worst evils, promotes immorality, and allows injustices to occur. In Gulliver’s Travels, Swift asks whether “civilization” is only an elaborate cover for the lowest forms of human behavior. Swift raises this question through satire, a literary form which he brings to perfection in Gulliver’s Travels. Following in the great tradition of classical satirists, Swift uses Gulliver’s Travels to point out the underlying ridiculousness of the society around him. In one section of Gulliver’s Travels, for example, Gulliver describes his native England to the king of a land of giants. Gulliver talks enthusiastically to the giant king about the class system, laws, constitution, military glory, and history of his homeland. After carefully listening to Gulliver’s speech, however, the king proceeds to point out the many shortcomings of Gulliver’s native land. Gulliver is unable to come up with adequate responses to the criticism. He can only stand there in embarrassed silence as the king eventually concludes that the institutions of the human world should be condemned.
 

The passage states that Swift wrote about a king in the land of giants in Gulliver’s Travels in order to

A
use an outsider’s perspective to satirize the ridiculousness of the society in which he lives.
B
contrast the giants’ monarchal system with the successful system in which Swift lives.
C
attempt to spark an open revolution lead by his readers.
D
satirize the way that people are quick to dismiss other cultures and customs.
E
emphasize how little Gulliver belongs in the land of the giants.
Question 2 Explanation: 
Answer choice (A) is correct because the passage spends a lot of time setting up Swift’s novel as a satirical criticism of his society. Answer choice (B) is incorrect because it suggests the opposite. Answer choice (D) might be tempting because of the use of the word “satirize,” but the passage doesn’t discuss Swift’s novel as a potential criticism of xenophobia and nationalism. While Swift is critical of his society, there is nothing in the passage to suggest that he’s trying to spark an open revolution, so answer choice (C) is incorrect. Answer choice (E) is too specific to the novel and doesn’t take Swift’s social and political intentions into account.
Question 3
Jonathan Swift’s dark view of humanity is expressed in his novel, Gulliver’s Travels. Swift believed that society, which is meant to protect people from inequality and injustice, actually encourages the worst evils, promotes immorality, and allows injustices to occur. In Gulliver’s Travels, Swift asks whether “civilization” is only an elaborate cover for the lowest forms of human behavior. Swift raises this question through satire, a literary form which he brings to perfection in Gulliver’s Travels. Following in the great tradition of classical satirists, Swift uses Gulliver’s Travels to point out the underlying ridiculousness of the society around him. In one section of Gulliver’s Travels, for example, Gulliver describes his native England to the king of a land of giants. Gulliver talks enthusiastically to the giant king about the class system, laws, constitution, military glory, and history of his homeland. After carefully listening to Gulliver’s speech, however, the king proceeds to point out the many shortcomings of Gulliver’s native land. Gulliver is unable to come up with adequate responses to the criticism. He can only stand there in embarrassed silence as the king eventually concludes that the institutions of the human world should be condemned.
 

Which of the following best describes the organization of the passage?

A
An exploration of the similarities and differences between two opposing ideas.
B
An introduction of an idea followed by specific examples that support it.
C
The presentation of an idea that is summarily refuted and discarded.
D
A series of steps that would aid in solving a problem.
E
A chronological arrangement of interrelated events.
Question 3 Explanation: 
Answer choice (B) is correct because the passage introduces Swift’s use of Gulliver’s Travels to satirize his society, then provides a specific example of this satirical criticism from the novel. There are no opposing ideas, as suggested by answer choice (A), nor is an idea presented that is then refuted, as suggested by answer choice (C). Answer choices (D) and (E) are similarly unsupported by the passage.
Question 4
Jonathan Swift’s dark view of humanity is expressed in his novel, Gulliver’s Travels. Swift believed that society, which is meant to protect people from inequality and injustice, actually encourages the worst evils, promotes immorality, and allows injustices to occur. In Gulliver’s Travels, Swift asks whether “civilization” is only an elaborate cover for the lowest forms of human behavior. Swift raises this question through satire, a literary form which he brings to perfection in Gulliver’s Travels. Following in the great tradition of classical satirists, Swift uses Gulliver’s Travels to point out the underlying ridiculousness of the society around him. In one section of Gulliver’s Travels, for example, Gulliver describes his native England to the king of a land of giants. Gulliver talks enthusiastically to the giant king about the class system, laws, constitution, military glory, and history of his homeland. After carefully listening to Gulliver’s speech, however, the king proceeds to point out the many shortcomings of Gulliver’s native land. Gulliver is unable to come up with adequate responses to the criticism. He can only stand there in embarrassed silence as the king eventually concludes that the institutions of the human world should be condemned.
 

Which of the following best explains why the author put “civilization” within quotation marks in the passage?

A
To signify that she is quoting another writer or speaker.
B
To acknowledge the fact that Swift coined the word.
C
To identify it as an important word that readers need to look up.
D
To highlight the fact that the characters of Gulliver’s Travels famously think the word is silly.
E
To emphasize Swift’s lack of confidence in civilization.
Question 4 Explanation: 
Answer choice (E) is correct because it is true to the tone of the passage. The point of the passage is that Swift is critical of civilization, so the author uses quotation marks to emphasize this feeling. Civilization is a fairly common word, so it is unlikely that the author is quoting anyone (A), or that Swift came up with the word (B). Based on the passage, the author seems to assume that readers know what the word means, so answer choice (C) is incorrect. Answer choice (D) might be tempting because it would make sense for the character’s in Swift’s satire criticizing civilization to treat the word as silly. However, the passage provides no evidence of this.
Question 5
Jonathan Swift’s dark view of humanity is expressed in his novel, Gulliver’s Travels. Swift believed that society, which is meant to protect people from inequality and injustice, actually encourages the worst evils, promotes immorality, and allows injustices to occur. In Gulliver’s Travels, Swift asks whether “civilization” is only an elaborate cover for the lowest forms of human behavior. Swift raises this question through satire, a literary form which he brings to perfection in Gulliver’s Travels. Following in the great tradition of classical satirists, Swift uses Gulliver’s Travels to point out the underlying ridiculousness of the society around him. In one section of Gulliver’s Travels, for example, Gulliver describes his native England to the king of a land of giants. Gulliver talks enthusiastically to the giant king about the class system, laws, constitution, military glory, and history of his homeland. After carefully listening to Gulliver’s speech, however, the king proceeds to point out the many shortcomings of Gulliver’s native land. Gulliver is unable to come up with adequate responses to the criticism. He can only stand there in embarrassed silence as the king eventually concludes that the institutions of the human world should be condemned.
 

Which of the following would the author of the passage most likely agree with?

A
Jonathan Swift’s criticisms of the English government were valid and noble.
B
The king of the land of giants in Gulliver’s Travels was a metaphor for the King of England.
C
Gulliver’s Travels, by Jonathan Swift, is one of the best satires ever written.
D
Not enough people still read Swift’s novel, Gulliver’s Travels.
E
The institutions of the human world are deserving of our condemnation.
Question 5 Explanation: 
Answer choice (C) is correct because it is the only option that is corroborated by the passage. The author writes that satire is “a literary form which [Swift] brings to perfection in Gulliver’s Travels.” Answer choices (A), (D), and (E) represent opinions that are not expressed by the author in the passage. Answer choice (B) is incorrect because it presents an incorrect understanding of the novel. In the context of the example provided in the passage, the king is a critic of English society, and therefore more closely represents Swift himself.
Question 6

Read the passages below; then answer questions 6–10.

Passage 1
Healthcare is a right that should be afforded to everyone in an evolved society. The amount of money you have should not dictate the kind of medical care you receive. Poor Americans are dying of diseases and maladies that wouldn’t even be a second thought for a rich American, who would be able to easily afford access to the medicine or care that would easily cure them. The longer that we as a country allow healthcare to be dictated by a person’s wealth, the more we reinforce the unethical privilege that the wealthy enjoy at the expense of the lives of poor Americans.

Passage 2
Healthcare is an important part of life and should be something that everyone works to earn. America is a capitalist society that rewards success, and to suddenly give healthcare to people who don’t even have a job would go against everything our country stands for. When the government starts to provide things for people who don’t have to work for them, America becomes a communist nation. People who don’t have adequate healthcare can change their circumstances by getting educated and starting a more prestigious career. Everyone in America should have the opportunity to succeed; that’s how our country thrives. If we give things away, what motivation does the average citizen have to work hard and make something of themselves?
 

The authors of these passages agree that

A
proper healthcare is important.
B
people should work hard for their healthcare.
C
everyone deserves comprehensive healthcare.
D
healthcare is a simple, straight-forward issue.
E
capitalism is not working for America.
Question 6 Explanation: 
Answer choice (A) is correct because both authors directly admit that healthcare is important. They don’t agree on much else, however. While the author of Passage 1 may agree with answer choices (C) and possible (E), the author of Passage 2 would not. Similarly, the author of Passage 2 would agree with answer choice (B), but the author of Passage 1 would not. It doesn’t seem like either of them would agree with answer choice (D).
Question 7
Passage 1
Healthcare is a right that should be afforded to everyone in an evolved society. The amount of money you have should not dictate the kind of medical care you receive. Poor Americans are dying of diseases and maladies that wouldn’t even be a second thought for a rich American, who would be able to easily afford access to the medicine or care that would easily cure them. The longer that we as a country allow healthcare to be dictated by a person’s wealth, the more we reinforce the unethical privilege that the wealthy enjoy at the expense of the lives of poor Americans.

Passage 2
Healthcare is an important part of life and should be something that everyone works to earn. America is a capitalist society that rewards success, and to suddenly give healthcare to people who don’t even have a job would go against everything our country stands for. When the government starts to provide things for people who don’t have to work for them, America becomes a communist nation. People who don’t have adequate healthcare can change their circumstances by getting educated and starting a more prestigious career. Everyone in America should have the opportunity to succeed; that’s how our country thrives. If we give things away, what motivation does the average citizen have to work hard and make something of themselves?
 

Which of the following statements best describes the relationship between the two passages?

A
Passage 1 is very specific in its evidence whereas Passage 2 remains very general.
B
Passage 1 is supported by objective data, while Passage 2 is supported by unsubstantiated opinion.
C
Passage 1 is very biased, while Passage 2 remains objective and factual.
D
Passage 1 focuses on what is causing a problem, while Passage 2 focuses on how to fix the problem.
E
Passage 1 and Passage 2 represent differing ideologies on a specific political issue.
Question 7 Explanation: 
Answer choice (E) is correct because Passage 1 and Passage 2 offer differing opinions on healthcare in America. Answer choice (D) is not supported by the passages. Neither of the passages is incredibly specific (A), nor are either of them supported by specific data (B). Both are biased, which refutes answer choice (C).
Question 8
Passage 1
Healthcare is a right that should be afforded to everyone in an evolved society. The amount of money you have should not dictate the kind of medical care you receive. Poor Americans are dying of diseases and maladies that wouldn’t even be a second thought for a rich American, who would be able to easily afford access to the medicine or care that would easily cure them. The longer that we as a country allow healthcare to be dictated by a person’s wealth, the more we reinforce the unethical privilege that the wealthy enjoy at the expense of the lives of poor Americans.

Passage 2
Healthcare is an important part of life and should be something that everyone works to earn. America is a capitalist society that rewards success, and to suddenly give healthcare to people who don’t even have a job would go against everything our country stands for. When the government starts to provide things for people who don’t have to work for them, America becomes a communist nation. People who don’t have adequate healthcare can change their circumstances by getting educated and starting a more prestigious career. Everyone in America should have the opportunity to succeed; that’s how our country thrives. If we give things away, what motivation does the average citizen have to work hard and make something of themselves?
 

Unlike the author of Passage 1, the author of Passage 2 mentions

A
how much good healthcare costs in America.
B
how many Americans currently have comprehensive healthcare.
C
specific forms of government.
D
the downfall of capitalism as the basis of the American economy.
E
the privilege that wealthy people have in America.
Question 8 Explanation: 
Answer choice (C) is correct because Passage 2 discusses both capitalism and communism. Neither of the passages addresses answer choices (A) and (B), while answer choices (D) and (E) represent opinions presented in Passage 1, not Passage 2.
Question 9
Passage 1
Healthcare is a right that should be afforded to everyone in an evolved society. The amount of money you have should not dictate the kind of medical care you receive. Poor Americans are dying of diseases and maladies that wouldn’t even be a second thought for a rich American, who would be able to easily afford access to the medicine or care that would easily cure them. The longer that we as a country allow healthcare to be dictated by a person’s wealth, the more we reinforce the unethical privilege that the wealthy enjoy at the expense of the lives of poor Americans.

Passage 2
Healthcare is an important part of life and should be something that everyone works to earn. America is a capitalist society that rewards success, and to suddenly give healthcare to people who don’t even have a job would go against everything our country stands for. When the government starts to provide things for people who don’t have to work for them, America becomes a communist nation. People who don’t have adequate healthcare can change their circumstances by getting educated and starting a more prestigious career. Everyone in America should have the opportunity to succeed; that’s how our country thrives. If we give things away, what motivation does the average citizen have to work hard and make something of themselves?
 

Which of the following problems mentioned in Passage 1 is directly addressed in Passage 2?

A
If we give everyone healthcare, people will have less motivation to work hard.
B
Healthcare costs way too much money in modern America.
C
Poor Americans are dying every day of curable diseases and treatable conditions.
D
Providing healthcare to only the wealthy prioritizes money over everything else.
E
America is a capitalist society, not a communist society.
Question 9 Explanation: 
Answer choice (D) is correct because Passage 1 points out that the American healthcare system prioritizes money over everything, while Passage 2 addresses this, citing it as an important aspect of capitalism. Answer choices (B) and (C) are addressed in Passage 1, but not in Passage 2. Answer choices (A) and (E) are addressed in Passage 2, but not in Passage 1.
Question 10
Passage 1
Healthcare is a right that should be afforded to everyone in an evolved society. The amount of money you have should not dictate the kind of medical care you receive. Poor Americans are dying of diseases and maladies that wouldn’t even be a second thought for a rich American, who would be able to easily afford access to the medicine or care that would easily cure them. The longer that we as a country allow healthcare to be dictated by a person’s wealth, the more we reinforce the unethical privilege that the wealthy enjoy at the expense of the lives of poor Americans.

Passage 2
Healthcare is an important part of life and should be something that everyone works to earn. America is a capitalist society that rewards success, and to suddenly give healthcare to people who don’t even have a job would go against everything our country stands for. When the government starts to provide things for people who don’t have to work for them, America becomes a communist nation. People who don’t have adequate healthcare can change their circumstances by getting educated and starting a more prestigious career. Everyone in America should have the opportunity to succeed; that’s how our country thrives. If we give things away, what motivation does the average citizen have to work hard and make something of themselves?
 

In the context of Passage 1, what does “afforded” most nearly mean?

A
Earned
B
Given
C
Arranged
D
Incurred
E
Supported
Question 10 Explanation: 
Answer choice (B) is correct because the author is saying that every citizen should be given healthcare as a right. None of the other answer choices work with the preposition that follows the word: “to.” Additionally, though, answer choice (A) more closely represents the ideas presented by the author of Passage 2, and answer choice (D) has a negative connotation that goes against the tone of the passage. Answer choices (C) and (E) simply don’t fit in the sentence.
Question 11

Read the passage below; then answer questions 11–16.

(1) Modern Americans tend to look back at the “founding fathers” and early presidents as infallible figures in history. (2) Just like politicians today, though, these men were flawed characters who don’t deserve to be remembered as great leaders. (3) One such man is Andrew Jackson, the seventh president of the United States, who served office from 1829 to 1837. (4) For many Americans, Jackson blends in with all the other early presidents, only standing out because he is the face on the $20 bill. (5) Jackson is remembered by others as an influential military leader prior to his presidency in the War of 1812, an important voice in the fight for the right to vote for all white males (not just landowners), and the first official president of the Democratic party. (6) To people who follow history closely, however, Jackson is best known by his nickname: “Indian killer.”

(7) In many ways, his military leadership in the Creek War, considered part of the War of 1812, helped shape the country Americans know today. (8) That is mostly because Jackson ordered his men to systematically kill Native American women and children in an effort to exterminate their tribes. (9) The decimation of Native Americans in the Creek War and other “battles” in the southeast paved the way for one of Jackson’s worst presidential maneuvers, the Trail of Tears. (10) By forcing Native Americans off their land, Jackson secured millions of acres of land for the US government and caused the deaths of thousands of Native Americans (not to mention the suffering and indignity of thousands more). (11) When we remember Andrew Jackson, we should remember him as “Indian Killer,” the worst president our country has ever seen.
 

The primary purpose of this passage is to

A
assert that one of America’s “founding fathers” was very flawed and not someone who should be held as a role model.
B
provide a profile of a great American hero.
C
argue that in order for America to become a great country, our “founding fathers” had to do a lot of bad stuff.
D
prove that all the “founding fathers” were not good people.
E
provide multiple perspectives on a troubled figure in American history.
Question 11 Explanation: 
Answer choice (A) is correct because while the passage mentions that everyone is flawed, the focus of the passage is specifically about Andrew Jackson. Answer choice (D) is too general to be accurate, and answer choices (C) and (E) reflect a much more conflicted opinion than is present in the passage. Answer choice (B) is directly refuted by the passage.
Question 12
(1) Modern Americans tend to look back at the “founding fathers” and early presidents as infallible figures in history. (2) Just like politicians today, though, these men were flawed characters who don’t deserve to be remembered as great leaders. (3) One such man is Andrew Jackson, the seventh president of the United States, who served office from 1829 to 1837. (4) For many Americans, Jackson blends in with all the other early presidents, only standing out because he is the face on the $20 bill. (5) Jackson is remembered by others as an influential military leader prior to his presidency in the War of 1812, an important voice in the fight for the right to vote for all white males (not just landowners), and the first official president of the Democratic party. (6) To people who follow history closely, however, Jackson is best known by his nickname: “Indian killer.”

(7) In many ways, his military leadership in the Creek War, considered part of the War of 1812, helped shape the country Americans know today. (8) That is mostly because Jackson ordered his men to systematically kill Native American women and children in an effort to exterminate their tribes. (9) The decimation of Native Americans in the Creek War and other “battles” in the southeast paved the way for one of Jackson’s worst presidential maneuvers, the Trail of Tears. (10) By forcing Native Americans off their land, Jackson secured millions of acres of land for the US government and caused the deaths of thousands of Native Americans (not to mention the suffering and indignity of thousands more). (11) When we remember Andrew Jackson, we should remember him as “Indian Killer,” the worst president our country has ever seen.
 

Which of the following sentences in the passage most directly supports the assertion made in the final sentence of the passage?

A
Sentence 1
B
Sentence 2
C
Sentence 4
D
Sentence 6
E
Sentence 8
Question 12 Explanation: 
Answer choice (E) is correct because it is the only option that directly discusses how Jackson’s actions caused the death of Native Americans. Answer choice (D) might be tempting because it uses the same term, “Indian Killer,” in reference to Andrew Jackson. However, Sentence 6 is more of a repetition of the same idea rather than a piece of evidence in support of it. Answer choices (A), (B), and (C) don’t reflect direct criticisms of Andrew Jackson, especially not on the level of what the author writes in the final sentence of the passage.
Question 13
(1) Modern Americans tend to look back at the “founding fathers” and early presidents as infallible figures in history. (2) Just like politicians today, though, these men were flawed characters who don’t deserve to be remembered as great leaders. (3) One such man is Andrew Jackson, the seventh president of the United States, who served office from 1829 to 1837. (4) For many Americans, Jackson blends in with all the other early presidents, only standing out because he is the face on the $20 bill. (5) Jackson is remembered by others as an influential military leader prior to his presidency in the War of 1812, an important voice in the fight for the right to vote for all white males (not just landowners), and the first official president of the Democratic party. (6) To people who follow history closely, however, Jackson is best known by his nickname: “Indian killer.”

(7) In many ways, his military leadership in the Creek War, considered part of the War of 1812, helped shape the country Americans know today. (8) That is mostly because Jackson ordered his men to systematically kill Native American women and children in an effort to exterminate their tribes. (9) The decimation of Native Americans in the Creek War and other “battles” in the southeast paved the way for one of Jackson’s worst presidential maneuvers, the Trail of Tears. (10) By forcing Native Americans off their land, Jackson secured millions of acres of land for the US government and caused the deaths of thousands of Native Americans (not to mention the suffering and indignity of thousands more). (11) When we remember Andrew Jackson, we should remember him as “Indian Killer,” the worst president our country has ever seen.
 

Which of the following numbered sentences from the passage best expresses an opinion rather than a fact?

A
Sentence 1
B
Sentence 3
C
Sentence 4
D
Sentence 5
E
Sentence 11
Question 13 Explanation: 
While the author’s bias seeps from each sentence in the passage, Sentence 11 is the only option that truly expresses an opinion, that Andrew Jackson should be remembered as America’s worst president. The other options are rather factual. While the author offers no evidence that Andrew Jackson “blends in with all the other early presidents,” this is a statement that readers are willing to reasonably accept as true without evidence.
Question 14
(1) Modern Americans tend to look back at the “founding fathers” and early presidents as infallible figures in history. (2) Just like politicians today, though, these men were flawed characters who don’t deserve to be remembered as great leaders. (3) One such man is Andrew Jackson, the seventh president of the United States, who served office from 1829 to 1837. (4) For many Americans, Jackson blends in with all the other early presidents, only standing out because he is the face on the $20 bill. (5) Jackson is remembered by others as an influential military leader prior to his presidency in the War of 1812, an important voice in the fight for the right to vote for all white males (not just landowners), and the first official president of the Democratic party. (6) To people who follow history closely, however, Jackson is best known by his nickname: “Indian killer.”

(7) In many ways, his military leadership in the Creek War, considered part of the War of 1812, helped shape the country Americans know today. (8) That is mostly because Jackson ordered his men to systematically kill Native American women and children in an effort to exterminate their tribes. (9) The decimation of Native Americans in the Creek War and other “battles” in the southeast paved the way for one of Jackson’s worst presidential maneuvers, the Trail of Tears. (10) By forcing Native Americans off their land, Jackson secured millions of acres of land for the US government and caused the deaths of thousands of Native Americans (not to mention the suffering and indignity of thousands more). (11) When we remember Andrew Jackson, we should remember him as “Indian Killer,” the worst president our country has ever seen.
 

Which of the following best describes the writer’s pattern of organization in the passage?

A
order of importance
B
explaining with examples
C
spatial order
D
chronological order
E
compare-contrast
Question 14 Explanation: 
In the passage, the author beings by describing what lots of people already think. He or she goes on to explain why that popular opinion is wrong, supporting ideas with examples. While the author’s final assertion is that Andrew Jackson was the worst US President, he/she doesn’t actual compare him to any other presidents, so choice (E) is incorrect. Considering the author’s placement of his/her main point as the final sentence, it’s hard to argue that the passage is organized by importance (A). The passage’s organization also has nothing to do with locations (C) or time (D). The correct answer is (B).
Question 15
(1) Modern Americans tend to look back at the “founding fathers” and early presidents as infallible figures in history. (2) Just like politicians today, though, these men were flawed characters who don’t deserve to be remembered as great leaders. (3) One such man is Andrew Jackson, the seventh president of the United States, who served office from 1829 to 1837. (4) For many Americans, Jackson blends in with all the other early presidents, only standing out because he is the face on the $20 bill. (5) Jackson is remembered by others as an influential military leader prior to his presidency in the War of 1812, an important voice in the fight for the right to vote for all white males (not just landowners), and the first official president of the Democratic party. (6) To people who follow history closely, however, Jackson is best known by his nickname: “Indian killer.”

(7) In many ways, his military leadership in the Creek War, considered part of the War of 1812, helped shape the country Americans know today. (8) That is mostly because Jackson ordered his men to systematically kill Native American women and children in an effort to exterminate their tribes. (9) The decimation of Native Americans in the Creek War and other “battles” in the southeast paved the way for one of Jackson’s worst presidential maneuvers, the Trail of Tears. (10) By forcing Native Americans off their land, Jackson secured millions of acres of land for the US government and caused the deaths of thousands of Native Americans (not to mention the suffering and indignity of thousands more). (11) When we remember Andrew Jackson, we should remember him as “Indian Killer,” the worst president our country has ever seen.
 

Which of the following is the best meaning of the word maneuvers as it is used in the passage?

A
strategic actions
B
terrible tragedies
C
smart decisions
D
lies he told
E
unsuccessful plans
Question 15 Explanation: 
The author is describing the Trail of Tears as a strategic action (A) here to emphasize that this was not an accident. While the author uses the word maneuvers to subtly nod to the audience that Jackson was not a good President, it is too severe to say the word itself means “terrible tragedies” (B). The author is clearly not happy about Jackson’s intentional actions, so (C) and (D) would not fit tonally. While the author is not happy about the Trail of Tears, it was successful in the author’s eyes because Jackson got rid of Native Americans like he wanted to, so (E) is incorrect.
Question 16
(1) Modern Americans tend to look back at the “founding fathers” and early presidents as infallible figures in history. (2) Just like politicians today, though, these men were flawed characters who don’t deserve to be remembered as great leaders. (3) One such man is Andrew Jackson, the seventh president of the United States, who served office from 1829 to 1837. (4) For many Americans, Jackson blends in with all the other early presidents, only standing out because he is the face on the $20 bill. (5) Jackson is remembered by others as an influential military leader prior to his presidency in the War of 1812, an important voice in the fight for the right to vote for all white males (not just landowners), and the first official president of the Democratic party. (6) To people who follow history closely, however, Jackson is best known by his nickname: “Indian killer.”

(7) In many ways, his military leadership in the Creek War, considered part of the War of 1812, helped shape the country Americans know today. (8) That is mostly because Jackson ordered his men to systematically kill Native American women and children in an effort to exterminate their tribes. (9) The decimation of Native Americans in the Creek War and other “battles” in the southeast paved the way for one of Jackson’s worst presidential maneuvers, the Trail of Tears. (10) By forcing Native Americans off their land, Jackson secured millions of acres of land for the US government and caused the deaths of thousands of Native Americans (not to mention the suffering and indignity of thousands more). (11) When we remember Andrew Jackson, we should remember him as “Indian Killer,” the worst president our country has ever seen.
 

What assumption best supports the author’s assertion that “The decimation of Native Americans in the Creek War and other ‘battles’ in the southeast paved the way for one of Jackson’s worst presidential maneuvers, The Trail of Tears”?

A
Jackson had been planning the Trail of Tears since the beginning of the Creek War.
B
Killing many Native American women and children in the Creek War and other “battles” made it easier for Jackson to later kick the Native Americans off their land in southeast America.
C
The Trail of Tears gave millions of acres of land to the US government and helped to shape the modern American southeast.
D
The Trail of Tears may have been a huge mistake, but it was the only recorded mistake that Jackson made during his presidency.
E
Andrew Jackson was president during the War of 1812 and used the war as a way to ethnically cleanse Native Americans.
Question 16 Explanation: 
As a military leader, Jackson ordered his men to kill Native American women and children in the Creek War. Later, as president, Jackson initiated the Indian Removal Act, which lead to the trail of tears. In the sentence mentioned in the question, the author asserts that the Creek War made it easier for Jackson to create the Trail of Tears. Only choice (B) addresses why the author would conclude this connection. Choices (A), (D), and (E) are directly refuted by the passage. Choice (E) is true, but the author states it later in the passage and it has no bearing on the logic the author uses here.
Question 17

Use the graph below to answer questions 17–18.

Which conclusion about summer enrollment at a local university in 2017 is best supported by the data presented in the graph above?

A
More students are taking Creative Writing than all the math classes combined
B
American History is the most popular class this summer
C
Finance is the least popular class this summer
D
Biology is the most popular science class
E
Most students are only taking the classes that are important
Question 17 Explanation: 
Mathematically, choices (A), (B), and (C) are incorrect. Answer choice (E) might be true, but the chart does not distinguish between important and unimportant classes, so this choice is a matter of unsubstantiated opinion. Biology has a higher percentage than physics and chemistry, so answer choice (D) is correct.
Question 18

Which department had the most students for their summer courses in 2017?

A
Business
B
History
C
Literature
D
Math
E
Science
Question 18 Explanation: 
The departments are shown below the pie chart. The history classes are marked in orange, and so you can add up the three classes marked with orange (European History, Civics, American History):

History: 8% + 6% + 9% = 23%

After adding up all departments you will see that history has the highest percentage. Next highest is science, at 22%.
Question 19

Read the passage below; then answer questions 19–21.

In the last few years, scientists in Australia have concluded that koalas sitting in trees aren’t just hanging out, as some people like to think. Using thermal imaging, scientists were able to determine that in hot weather, koalas climb down lower on their trees, where the tree is significantly cooler. When the weather turns cold, they move higher up the tree. This study was done as part of a larger environmental study researching the effects of climate change on animals indigenous to Australia. Scientists fear that koalas may have to find a more effective way to cool themselves as climate change continues to affect their environments.
 

Which of the following words, if substituted for the word “indigenous” in the passage, would introduce the LEAST change in the meaning of the sentence?

A
Domestic
B
Original
C
Native
D
Primitive
E
Foreign
Question 19 Explanation: 
Answer choice (C) is correct because it most nearly maintains the meaning that koalas are natural inhabitants of Australia. Answer choices (A) and (B) might be tempting because they are very near the right meaning, but they change the meaning of the sentence slightly. “Domestic” means “of or relating to the home,” while “original” means “belonging or pertaining to the beginning of something.” Neither of these is quite right. Answer choice (E) is an antonym for “indigenous,” and answer choice (D) has the wrong connotation for the sentence.
Question 20
In the last few years, scientists in Australia have concluded that koalas sitting in trees aren’t just hanging out, as some people like to think. Using thermal imaging, scientists were able to determine that in hot weather, koalas climb down lower on their trees, where the tree is significantly cooler. When the weather turns cold, they move higher up the tree. This study was done as part of a larger environmental study researching the effects of climate change on animals indigenous to Australia. Scientists fear that koalas may have to find a more effective way to cool themselves as climate change continues to affect their environments.
 

According to the final sentence of the passage, climate change

A
will likely make it easier for koalas to find a cool spot.
B
has already caused koalas to be an endangered species.
C
is a problem that koalas are already well-equipped to face.
D
is not a threat to animals like the koala.
E
may make it difficult for koalas to use this method of temperature adaptation.
Question 20 Explanation: 
Answer choice (E) is correct because the final sentence expresses the fear that climate change will make things more difficult for koalas. While climate change may already be affecting koalas, they are not an endangered species (B). Answer choices (C) and (D) are incorrect because they proport that we don’t have to worry about how climate change will affect koalas, a view directly refuted by the final sentence in the passage. Answer choice (A) is incorrect because it operates on a logical and fundamental misunderstanding of climate change.
Question 21
In the last few years, scientists in Australia have concluded that koalas sitting in trees aren’t just hanging out, as some people like to think. Using thermal imaging, scientists were able to determine that in hot weather, koalas climb down lower on their trees, where the tree is significantly cooler. When the weather turns cold, they move higher up the tree. This study was done as part of a larger environmental study researching the effects of climate change on animals indigenous to Australia. Scientists fear that koalas may have to find a more effective way to cool themselves as climate change continues to affect their environments.
 

In the passage, the author is primarily concerned with

A
animals that find unique ways to survive in their environment
B
the habits of koalas in a changing climate.
C
the effects of climate change on Australian animals.
D
what scientists can do to help koalas adapt to climate change.
E
the causes of climate change and how we can combat it.
Question 21 Explanation: 
Answer choice (B) is the correct answer because the passage discusses koalas specifically, and how a habit of koalas may be affected by climate change. Answer choices (A) and (C) are too general, as the passage focuses only on koalas. Answer choices (D) and (E) are focused too narrowly on preventing climate change and how koalas can adapt to it, two ideas that are not broached in the passage.
Question 22

Read the passages below; then answer questions 22–26.

Passage 1
Many teachers across the country are noticing the importance of film in the modern classroom. No longer is putting a movie on meant as an invitation for students to check out while the teacher does work at their desk. Films are a versatile medium, and can function as textbooks, pieces of literature, or pieces of art that can bring a new dimension to a student’s learning. Modern education standards (including the much-maligned Common Core Standards) recognize visual media as an important learning tool and require it as part of a students’ education. Not all teachers are comfortable teaching with films, however, most likely because they are used to the stigma of showing movies as a substitute for actual teaching. The teachers leading the charge for the effective use of film in the classroom know that teaching a film effectively means preparing lessons with the same amount of time and effort as any other days. When teachers allow themselves to approach teaching with a film with the same sense of care that they treat teaching with a short story, a novel, or a textbook, the students will be the ones who reap the benefits.

Passage 2
Picture a history classroom that has been studying World War II for the past two weeks, most recently focusing on the Invasion of Normandy Beach (D-Day). A traditional teacher might simply shut the lights and throw on the first 20 minutes of Steven Spielberg’s “Saving Private Ryan.” Students put their heads down, go on their phones, or pass notes throughout the lesson; nothing is gained. Or the teacher could spend a class period analyzing the same 20-minute scene with the class, looking at how the director, Steven Spielberg, builds anxiety in the beginning, how he creates the chaos of the beach, and how he gets the audience to care for characters they haven’t even met yet. Students would get much more out of this second approach and would be able to appreciate the carnage and the pandemonium that soldiers experienced on the beaches of Normandy. When teachers try to get students to understand a novel, they don’t just read the novel aloud straight through and then move on to the next lesson. They use class time to do in-depth analysis of the novel. Why should it be any different when a teacher uses a film? Teachers with the second approach are revolutionizing what it means to watch a movie in class and taking advantage of a great learning tool.
 

As used in Passage 1, “versatile” most nearly means

A
Something that has a strong rhythm
B
Something that is grounded in reality
C
Something that is easy to be misinterpreted
D
Something that can be used in many different ways
E
Something that has both video and audio
Question 22 Explanation: 
Answer choice (D) is the correct answer because the passage focuses on how films can be used for many different functions in the classroom. Answer choice (E) might be tempting because it is accurate to the denotative meaning of the word, but it doesn’t make sense in the context of the sentence and passage. Answer choices (A), (B), and (C) don’t accurately represent the meaning of the word “versatile.”
Question 23
Passage 1
Many teachers across the country are noticing the importance of film in the modern classroom. No longer is putting a movie on meant as an invitation for students to check out while the teacher does work at their desk. Films are a versatile medium, and can function as textbooks, pieces of literature, or pieces of art that can bring a new dimension to a student’s learning. Modern education standards (including the much-maligned Common Core Standards) recognize visual media as an important learning tool and require it as part of a students’ education. Not all teachers are comfortable teaching with films, however, most likely because they are used to the stigma of showing movies as a substitute for actual teaching. The teachers leading the charge for the effective use of film in the classroom know that teaching a film effectively means preparing lessons with the same amount of time and effort as any other days. When teachers allow themselves to approach teaching with a film with the same sense of care that they treat teaching with a short story, a novel, or a textbook, the students will be the ones who reap the benefits.

Passage 2
Picture a history classroom that has been studying World War II for the past two weeks, most recently focusing on the Invasion of Normandy Beach (D-Day). A traditional teacher might simply shut the lights and throw on the first 20 minutes of Steven Spielberg’s “Saving Private Ryan.” Students put their heads down, go on their phones, or pass notes throughout the lesson; nothing is gained. Or the teacher could spend a class period analyzing the same 20-minute scene with the class, looking at how the director, Steven Spielberg, builds anxiety in the beginning, how he creates the chaos of the beach, and how he gets the audience to care for characters they haven’t even met yet. Students would get much more out of this second approach and would be able to appreciate the carnage and the pandemonium that soldiers experienced on the beaches of Normandy. When teachers try to get students to understand a novel, they don’t just read the novel aloud straight through and then move on to the next lesson. They use class time to do in-depth analysis of the novel. Why should it be any different when a teacher uses a film? Teachers with the second approach are revolutionizing what it means to watch a movie in class and taking advantage of a great learning tool.
 

Which statement, if true, would most weaken the implied argument in Passage 2 regarding “Saving Private Ryan”?

A
Steven Spielberg has since openly admitted to fabricating much of what is shown in the opening scene of his film, “Saving Private Ryan.”
B
Studies show that students are more likely to empathize with film characters than people they read about in a textbook.
C
Teachers surveyed report that “Saving Private Ryan” is the film that they have analyzed has the greatest effect on students when compared to all other films shown in class.
D
Most school districts are beginning to embrace the use of films in the classroom, and no longer viewing them “a waste of time.”
E
At the premiere of “Saving Private Ryan,” many veterans reportedly had to leave the theater during the scene in question because of how realistic it was.
Question 23 Explanation: 
Answer choice (A) is correct because the inherent conceit of the example in Passage 2 is that students will learn about what D-Day was like by watching the film. If Spielberg confesses that the film is fabricated, then students will not learn about the actual military mission from watching it in class. Each of the other answer choices would support the use of this film in the classroom.
Question 24
Passage 1
Many teachers across the country are noticing the importance of film in the modern classroom. No longer is putting a movie on meant as an invitation for students to check out while the teacher does work at their desk. Films are a versatile medium, and can function as textbooks, pieces of literature, or pieces of art that can bring a new dimension to a student’s learning. Modern education standards (including the much-maligned Common Core Standards) recognize visual media as an important learning tool and require it as part of a students’ education. Not all teachers are comfortable teaching with films, however, most likely because they are used to the stigma of showing movies as a substitute for actual teaching. The teachers leading the charge for the effective use of film in the classroom know that teaching a film effectively means preparing lessons with the same amount of time and effort as any other days. When teachers allow themselves to approach teaching with a film with the same sense of care that they treat teaching with a short story, a novel, or a textbook, the students will be the ones who reap the benefits.

Passage 2
Picture a history classroom that has been studying World War II for the past two weeks, most recently focusing on the Invasion of Normandy Beach (D-Day). A traditional teacher might simply shut the lights and throw on the first 20 minutes of Steven Spielberg’s “Saving Private Ryan.” Students put their heads down, go on their phones, or pass notes throughout the lesson; nothing is gained. Or the teacher could spend a class period analyzing the same 20-minute scene with the class, looking at how the director, Steven Spielberg, builds anxiety in the beginning, how he creates the chaos of the beach, and how he gets the audience to care for characters they haven’t even met yet. Students would get much more out of this second approach and would be able to appreciate the carnage and the pandemonium that soldiers experienced on the beaches of Normandy. When teachers try to get students to understand a novel, they don’t just read the novel aloud straight through and then move on to the next lesson. They use class time to do in-depth analysis of the novel. Why should it be any different when a teacher uses a film? Teachers with the second approach are revolutionizing what it means to watch a movie in class and taking advantage of a great learning tool.
 

When the author discusses how teachers use novels in the classroom in Passage 2, what type of figurative language is she using?

A
Metaphor
B
Simile
C
Analogy
D
Hyperbole
E
Symbolism
Question 24 Explanation: 
Answer choice (C) is correct because the author is making a comparison between two situations: teaching a film and teaching a novel. It might be tempting to choose answer choice (A) or answer choice (B). The difference between an analogy and a metaphor can be difficult to pin down, but a metaphor would be a more direct comparison rather than a similar example that is presented for comparison. A metaphor in this passage might look like this: “teachers who don’t analyze a film with their students are sending them out of a plane without a chute.” Answer choices (D) and (E) simply don’t fit.
Question 25
Passage 1
Many teachers across the country are noticing the importance of film in the modern classroom. No longer is putting a movie on meant as an invitation for students to check out while the teacher does work at their desk. Films are a versatile medium, and can function as textbooks, pieces of literature, or pieces of art that can bring a new dimension to a student’s learning. Modern education standards (including the much-maligned Common Core Standards) recognize visual media as an important learning tool and require it as part of a students’ education. Not all teachers are comfortable teaching with films, however, most likely because they are used to the stigma of showing movies as a substitute for actual teaching. The teachers leading the charge for the effective use of film in the classroom know that teaching a film effectively means preparing lessons with the same amount of time and effort as any other days. When teachers allow themselves to approach teaching with a film with the same sense of care that they treat teaching with a short story, a novel, or a textbook, the students will be the ones who reap the benefits.

Passage 2
Picture a history classroom that has been studying World War II for the past two weeks, most recently focusing on the Invasion of Normandy Beach (D-Day). A traditional teacher might simply shut the lights and throw on the first 20 minutes of Steven Spielberg’s “Saving Private Ryan.” Students put their heads down, go on their phones, or pass notes throughout the lesson; nothing is gained. Or the teacher could spend a class period analyzing the same 20-minute scene with the class, looking at how the director, Steven Spielberg, builds anxiety in the beginning, how he creates the chaos of the beach, and how he gets the audience to care for characters they haven’t even met yet. Students would get much more out of this second approach and would be able to appreciate the carnage and the pandemonium that soldiers experienced on the beaches of Normandy. When teachers try to get students to understand a novel, they don’t just read the novel aloud straight through and then move on to the next lesson. They use class time to do in-depth analysis of the novel. Why should it be any different when a teacher uses a film? Teachers with the second approach are revolutionizing what it means to watch a movie in class and taking advantage of a great learning tool.
 

Which of the following statements best describes the relationship between the two passages?

A
The two passages present two different opinions on the same topic.
B
Passage 1 presents a problem and its causes, while Passage 2 discusses the solutions to that problem.
C
Passage 1 is supported by objective data, while Passage 2 is supported by unsubstantiated opinion.
D
Passage 2 is very specific in its evidence whereas Passage 1 remains very general.
E
Passage 1 is very biased, while Passage 2 remains objective and factual.
Question 25 Explanation: 
Answer choice (D) is correct because the two passages offer a similar opinion, with Passage 1 focusing on the general idea that watching films in the classroom can be beneficial to students, and Passage 2 presenting a very specific situation in which students can benefit from watching a film in the classroom. None of the other answer choices reflects the nuanced difference between the two passages.
Question 26
Passage 1
Many teachers across the country are noticing the importance of film in the modern classroom. No longer is putting a movie on meant as an invitation for students to check out while the teacher does work at their desk. Films are a versatile medium, and can function as textbooks, pieces of literature, or pieces of art that can bring a new dimension to a student’s learning. Modern education standards (including the much-maligned Common Core Standards) recognize visual media as an important learning tool and require it as part of a students’ education. Not all teachers are comfortable teaching with films, however, most likely because they are used to the stigma of showing movies as a substitute for actual teaching. The teachers leading the charge for the effective use of film in the classroom know that teaching a film effectively means preparing lessons with the same amount of time and effort as any other days. When teachers allow themselves to approach teaching with a film with the same sense of care that they treat teaching with a short story, a novel, or a textbook, the students will be the ones who reap the benefits.

Passage 2
Picture a history classroom that has been studying World War II for the past two weeks, most recently focusing on the Invasion of Normandy Beach (D-Day). A traditional teacher might simply shut the lights and throw on the first 20 minutes of Steven Spielberg’s “Saving Private Ryan.” Students put their heads down, go on their phones, or pass notes throughout the lesson; nothing is gained. Or the teacher could spend a class period analyzing the same 20-minute scene with the class, looking at how the director, Steven Spielberg, builds anxiety in the beginning, how he creates the chaos of the beach, and how he gets the audience to care for characters they haven’t even met yet. Students would get much more out of this second approach and would be able to appreciate the carnage and the pandemonium that soldiers experienced on the beaches of Normandy. When teachers try to get students to understand a novel, they don’t just read the novel aloud straight through and then move on to the next lesson. They use class time to do in-depth analysis of the novel. Why should it be any different when a teacher uses a film? Teachers with the second approach are revolutionizing what it means to watch a movie in class and taking advantage of a great learning tool.
 

Which of the following statements from Passage 1 is NOT directly supported by the author’s “Saving Private Ryan” example in Passage 2?

A
“Many teachers across the country are noticing the importance of film in the modern classroom."
B
“No longer is putting a movie on meant as an invitation for students to check out while the teacher does work at their desk."
C
“Films are a versatile medium, and can function as textbooks, pieces of literature, or pieces of art that can bring a new dimension to a student’s learning.”
D
“Modern education standards (including the much-maligned Common Core Standards) recognize visual media as an important learning tool and require it as part of a students’ education.”
E
"Not all teachers are comfortable teaching with films, however, most likely because they are used to the stigma of showing movies as a substitute for actual teaching.”
Question 26 Explanation: 
Answer choice (E) is correct because, while the example in Passage 2 functions as an assuagement of the concerns expressed in the sentence, it does not directly support it. The example provided in Passage 2 addresses the importance of film (A), engages students (B), uses films as a versatile medium (C), and keeps in line with modern educations standards (D).
Question 27

Use following table of contents from a biology textbook below to answer questions 27–30.

The use of the word “foundations” in the title for Unit 1 indicates that

A
the first unit simply contains concepts to warm up the minds of students before getting to the hard chapters.
B
the teachings in Unit 1 will be essential to understanding everything else that will be taught from the book for the duration of the school year.
C
the first unit can be skipped so that the teacher has time to get through the entire textbook before the end of the year.
D
students must have a foundational understanding of reading, writing, and arithmetic in order to succeed in this class.
E
the first unit will review the concepts taught in previous science classes without adding any new material.
Question 27 Explanation: 
The word “Foundations” indicates the underlying principles that will help students make sense of all of the material that follows, which makes answer choice (B) correct. The word communicates that the students must immediately engage in grasping the concepts taught in the first unit if they are to succeed in understanding the later units. Answer choice (D) is incorrect because the chapter titles in the first unit show that the foundations of reading, writing, and arithmetic are not the foundations to which the unit title is referring. Each of the other answer choices represents a misunderstanding of the word “foundations.”
Question 28

Which unit would likely help students to understand why children often resemble their parents in their appearance?

A
Unit 1
B
Unit 2
C
Unit 3
D
Unit 4
E
Unit 5
Question 28 Explanation: 
The title of Unit 3 includes the word “Genetics,” a word that students have learned in basic science classes before taking Biology. Some of the chapter titles in the unit refer to “genes,” and the title for Chapter 12 includes “Inheritance Patterns,” indicating that Unit 3 would be the best place to find out about children inheriting physical traits from their parents.
Question 29

When one examines the unit title and the chapter titles for Unit 6, what conclusion does NOT seem evident?

A
Microbes, protists, and fungi must be biologically related in some way.
B
Fungi are probably a type of protist.
C
Viruses and bacteria are probably two types of microbes.
D
Though many people do not use the term “protists,” they must represent an important category of organisms in biology.
E
These chapters are about smaller and simpler forms of life than the life forms in the later units.
Question 29 Explanation: 
The conclusion in choice (A) is reasonable since the three are grouped together into one unit. The conclusion is choice (C) is less obvious, but it is reasonable when one compares the unit title with the chapter title. Choice (D) is reasonably evident, since the textbook chooses to devote an entire chapter to protists. Choice (E) is also somewhat obvious to those who know that bacteria and viruses are tiny organisms. Choice (B) is not likely, since the unit title puts “protists” and “fungi” in a list of equals, and since fungi are not discussed as a subcategory of protists in the chapter on protists.
Question 30

The organization method for the second half of the book (Units 6 through 10) can be most accurately described as

A
going from lesser life forms to greater life forms
B
going from tiny life forms to the biggest life forms
C
going from later life forms to earlier life forms
D
going from complicated life forms that are hard to understand to the life forms we see every day
E
a random catalogue of the major categories of life forms
Question 30 Explanation: 
From previous science courses, a student will understand that humans are the most complex of life forms, with animals a bit lower (lacking complex language), and plants a bit lower (lacking visible emotions, and microscopic life forms being less complex than plants. Therefore, choice (A) is the best description among the choices given for the arrangement of the second half of the units. Since many mammals are larger than humans, choice (B) is not correct.
Question 31

Read the passages below; then answer questions 31–34.

Passage 1
Whether people like to admit it or not, the urge to belong to a group is a major motivation behind much of what they do. It is important to be an individual, but being an individual at the expense of alienating one’s self from everyone else can be harmful to a person’s mental and psychological health. Experts like to coach children to be themselves, to not conform, but true non-conformity is dangerous. Everyone needs to sacrifice some individuality in order to exist within society. What 13-year-old, if given complete choice, would choose to go to school every day? If teenagers were to give in to their individual desires at all times, our society would regress into chaos and madness. This “freedom” would also lead to more rampant loneliness, and subsequently a rising suicide rate. Individualism is great to a point; giving up some of our individuality to engage in our society, however, is the best way to improve ourselves as individuals and as a group.

Passage 2
The worst thing that an individual can do is conform to a group or society. When people conform, they lose their identity. Critics of this idea often bring up the straw man argument that we have to conform to society in order to truly exist within it. While this is true, it is also a flawed argument. By getting a job, paying taxes, or doing other mundane activities that you don’t want to do, you are not truly conforming because they are choices that benefit you. True individualism isn’t selfish myopia. True individualism is making the choices that you want to make without worry of outside pressures or what other people will think. Even if you don’t have children, it is in your better interest to fund education with your taxes. Proper education decreases crime and helps a community make progress. Our individualism is the most important thing in the world, and we should be free to make our own choices, even if those choices happen to align with the choices of another. Conformity happens when people feel forced to align their choices with the choices of others, and that kind of decision-making kills the individual.
 

With which of the following statements would the authors of both passages agree?

A
People need to engage in social constructs to be happy.
B
Paying taxes is not conformity if your taxes benefit you and/or your interests.
C
Anytime you do something you don’t want to do, you’re conforming, regardless of how it benefits you.
D
Individuality will only lead to chaos and mayhem.
E
Individual freedom is a necessity.
Question 31 Explanation: 
Answer choice (E) is correct because even the author of Passage 1 agrees that individuality, in moderation, is important. Each of the other choices reflects an idea that only one author would agree with; only the author of Passage 1 would agree with answer choices (A), (C), and (D), while only the author of Passage 2 would agree with answer choice (B).
Question 32
Passage 1
Whether people like to admit it or not, the urge to belong to a group is a major motivation behind much of what they do. It is important to be an individual, but being an individual at the expense of alienating one’s self from everyone else can be harmful to a person’s mental and psychological health. Experts like to coach children to be themselves, to not conform, but true non-conformity is dangerous. Everyone needs to sacrifice some individuality in order to exist within society. What 13-year-old, if given complete choice, would choose to go to school every day? If teenagers were to give in to their individual desires at all times, our society would regress into chaos and madness. This “freedom” would also lead to more rampant loneliness, and subsequently a rising suicide rate. Individualism is great to a point; giving up some of our individuality to engage in our society, however, is the best way to improve ourselves as individuals and as a group.

Passage 2
The worst thing that an individual can do is conform to a group or society. When people conform, they lose their identity. Critics of this idea often bring up the straw man argument that we have to conform to society in order to truly exist within it. While this is true, it is also a flawed argument. By getting a job, paying taxes, or doing other mundane activities that you don’t want to do, you are not truly conforming because they are choices that benefit you. True individualism isn’t selfish myopia. True individualism is making the choices that you want to make without worry of outside pressures or what other people will think. Even if you don’t have children, it is in your better interest to fund education with your taxes. Proper education decreases crime and helps a community make progress. Our individualism is the most important thing in the world, and we should be free to make our own choices, even if those choices happen to align with the choices of another. Conformity happens when people feel forced to align their choices with the choices of others, and that kind of decision-making kills the individual.
 

Why is the specific mention of a 13-year old effective in supporting the main idea of Passage 1?

A
It uses specific data to support the author’s argument.
B
It connects the readers to a shared history of what it was like to be an adolescent teenager in order to support the author’s argument.
C
It provides a specific anecdote that supports the author’s argument.
D
It shows the readers that the author is a professional who is educated on the topic.
E
It provides a counter-argument that refutes a common argument made by the opposition.
Question 32 Explanation: 
Answer choice (B) is correct because the author is trying to connect her argument to something to which all readers can relate: being a 13-year-old. Answer choice (C) might be tempting, but a hypothetical example isn’t an anecdote. If the author were to write something like, “imagine my 13-year old son, Billy, having the freedom to…”, then that would be an anecdote. The author is not making a counter-argument (E), nor is she providing specific data (A). While this is a good example, just the example itself doesn’t prove to readers that the author is a professional who is educated on the topic, so answer choice (D) is also incorrect.
Question 33
Passage 1
Whether people like to admit it or not, the urge to belong to a group is a major motivation behind much of what they do. It is important to be an individual, but being an individual at the expense of alienating one’s self from everyone else can be harmful to a person’s mental and psychological health. Experts like to coach children to be themselves, to not conform, but true non-conformity is dangerous. Everyone needs to sacrifice some individuality in order to exist within society. What 13-year-old, if given complete choice, would choose to go to school every day? If teenagers were to give in to their individual desires at all times, our society would regress into chaos and madness. This “freedom” would also lead to more rampant loneliness, and subsequently a rising suicide rate. Individualism is great to a point; giving up some of our individuality to engage in our society, however, is the best way to improve ourselves as individuals and as a group.

Passage 2
The worst thing that an individual can do is conform to a group or society. When people conform, they lose their identity. Critics of this idea often bring up the straw man argument that we have to conform to society in order to truly exist within it. While this is true, it is also a flawed argument. By getting a job, paying taxes, or doing other mundane activities that you don’t want to do, you are not truly conforming because they are choices that benefit you. True individualism isn’t selfish myopia. True individualism is making the choices that you want to make without worry of outside pressures or what other people will think. Even if you don’t have children, it is in your better interest to fund education with your taxes. Proper education decreases crime and helps a community make progress. Our individualism is the most important thing in the world, and we should be free to make our own choices, even if those choices happen to align with the choices of another. Conformity happens when people feel forced to align their choices with the choices of others, and that kind of decision-making kills the individual.
 

Which of the following statements most sounds like a response that the author of Passage 2 would have to the hypothetical example used by the author in Passage 1?

A
When a teenager decides to go to school, he or she is conforming to what society wants rather than what he or she wants.
B
People can only truly become individuals when they are adults and can understand the full impact of their choices.
C
Going to school benefits the individual, and most teenagers would choose to be educated rather than not, even if they don’t necessarily want to go to school.
D
Even individuals need to conform to society in order to succeed and flourish.
E
Individualism can only come when a person rejects everything undesirable and only does at any given moment what he or she wants to do.
Question 33 Explanation: 
The author of Passage 2 argues that deciding to do something you don’t want to do isn’t necessarily conformity, as long as it helps you fulfill your individuality. Only answer choice (C) reflects this opinion. Answer choice (D) might be tempting, but since the author of Passage 2 wouldn’t consider a teenager going to school as “conforming,” it doesn’t work. The other answer choices either express an opinion only held by the author of Passage 1 ((A) and (E)), or an opinion held by neither of the authors (B).
Question 34
Passage 1
Whether people like to admit it or not, the urge to belong to a group is a major motivation behind much of what they do. It is important to be an individual, but being an individual at the expense of alienating one’s self from everyone else can be harmful to a person’s mental and psychological health. Experts like to coach children to be themselves, to not conform, but true non-conformity is dangerous. Everyone needs to sacrifice some individuality in order to exist within society. What 13-year-old, if given complete choice, would choose to go to school every day? If teenagers were to give in to their individual desires at all times, our society would regress into chaos and madness. This “freedom” would also lead to more rampant loneliness, and subsequently a rising suicide rate. Individualism is great to a point; giving up some of our individuality to engage in our society, however, is the best way to improve ourselves as individuals and as a group.

Passage 2
The worst thing that an individual can do is conform to a group or society. When people conform, they lose their identity. Critics of this idea often bring up the straw man argument that we have to conform to society in order to truly exist within it. While this is true, it is also a flawed argument. By getting a job, paying taxes, or doing other mundane activities that you don’t want to do, you are not truly conforming because they are choices that benefit you. True individualism isn’t selfish myopia. True individualism is making the choices that you want to make without worry of outside pressures or what other people will think. Even if you don’t have children, it is in your better interest to fund education with your taxes. Proper education decreases crime and helps a community make progress. Our individualism is the most important thing in the world, and we should be free to make our own choices, even if those choices happen to align with the choices of another. Conformity happens when people feel forced to align their choices with the choices of others, and that kind of decision-making kills the individual.
 

Which of the following statements best describes the relationship between the two passages?

A
Passage 1 presents a problem and its causes, while Passage 2 discusses the solutions to that problem.
B
Passage 1 is supported by objective data, while Passage 2 is supported by unsubstantiated opinion.
C
Passage 2 is very specific in its evidence whereas Passage 1 remains very general.
D
The two passages present two different opinions on the same topic.
E
Passage 1 is very biased, while Passage 2 remains objective and factual.
Question 34 Explanation: 
Answer choice (D) is correct because even though the two passages have some common ground, they ultimately have opposing views on individualism and conformity. Neither of the passages is objective ((B) and (E)), nor is either of them incredibly specific (C). Answer choice (A) is incorrect because it implies that the two authors are on the same page, but simply writing about different aspects of a problem (problem/solution), which is not evident in the passages.
Question 35

Read the passage below; then answer questions 35–38.

My family used to take yearly trips to Florida to go on vacation and visit my grandmother. I used to love these trips because the warm weather, pools, and beaches were a big change from what I was used to in Connecticut. I woke up one morning, the day we were set to leave for one of these vacations, to find my parents already out of bed. As I walked downstairs, I could feel that the air was different than I expected. It wasn’t light and carefree; it was heavy and burdened. That’s when I saw my parents talking with my aunt. I couldn’t figure out why she was there, but they all looked very upset. We didn’t go on vacation that day. My grandmother had died. I hated my parents for inviting all of our family over to laugh and tell stories. Why was everyone so happy? It took a long time for me to realize that that was just how my family dealt with death.
 

The primary purpose of this passage is to

A
inform the readers on the most effective way to cope with grief.
B
entertain the readers with an amusing anecdote about the writer’s childhood.
C
present two opposing viewpoints of the same issue.
D
tell the readers a story of an important moment in the writer’s life.
E
persuade readers to be more tactful in how they introduce their children to death.
Question 35 Explanation: 
Answer choice (D) is correct because the passage is not meant to teach the audience a lesson or take a specific stance. Instead, the writer is just telling the audience something about himself using an important story from his childhood.
Question 36
My family used to take yearly trips to Florida to go on vacation and visit my grandmother. I used to love these trips because the warm weather, pools, and beaches were a big change from what I was used to in Connecticut. I woke up one morning, the day we were set to leave for one of these vacations, to find my parents already out of bed. As I walked downstairs, I could feel that the air was different than I expected. It wasn’t light and carefree; it was heavy and burdened. That’s when I saw my parents talking with my aunt. I couldn’t figure out why she was there, but they all looked very upset. We didn’t go on vacation that day. My grandmother had died. I hated my parents for inviting all of our family over to laugh and tell stories. Why was everyone so happy? It took a long time for me to realize that that was just how my family dealt with death.
 

What does the sentence, “I hated my parents for inviting all of our family over to laugh and tell stories” tell us about the narrator?

A
The narrator was mad that she couldn’t go on vacation
B
The narrator was used to experiencing death
C
The narrator misunderstood the behavior of her family
D
The narrator had a bad relationship with her parents
E
The narrator was upset that her family didn’t like her grandmother
Question 36 Explanation: 
Based on the sentence that follows, the narrator didn’t realize how sad her family really was. This makes Choice (C) correct, but also makes Choice (E) incorrect. Choice (A) may be true, but there is nothing to support it in the passage. The details in the passage pretty directly refute choices (B) and (D).
Question 37
My family used to take yearly trips to Florida to go on vacation and visit my grandmother. I used to love these trips because the warm weather, pools, and beaches were a big change from what I was used to in Connecticut. I woke up one morning, the day we were set to leave for one of these vacations, to find my parents already out of bed. As I walked downstairs, I could feel that the air was different than I expected. It wasn’t light and carefree; it was heavy and burdened. That’s when I saw my parents talking with my aunt. I couldn’t figure out why she was there, but they all looked very upset. We didn’t go on vacation that day. My grandmother had died. I hated my parents for inviting all of our family over to laugh and tell stories. Why was everyone so happy? It took a long time for me to realize that that was just how my family dealt with death.
 

What type of figurative language can you identify in the following quote from the passage?

“As I walked downstairs, I could feel that the air was different than I expected. It wasn’t light and carefree; it was heavy and burdened.”

A
Imagery
B
Simile
C
Understatement
D
Onomatopoeia
E
Alliteration
Question 37 Explanation: 
Answer choice (A) is correct because the writer provides details about the feel of the air that morning, which appeals to the reader’s sense of touch. Answer choice (B) might be tempting because the quote does include a near comparison, but it definitely isn’t a simile. Answer choice (C) is incorrect because the writer isn’t intentionally underplaying what happened, while answer choice (D) is incorrect because the writer doesn’t describe a sound in the quote. There is also no alliteration, or repetition of a sound at the beginning of words, in the quote, which means answer choice (E) is incorrect.
Question 38
My family used to take yearly trips to Florida to go on vacation and visit my grandmother. I used to love these trips because the warm weather, pools, and beaches were a big change from what I was used to in Connecticut. I woke up one morning, the day we were set to leave for one of these vacations, to find my parents already out of bed. As I walked downstairs, I could feel that the air was different than I expected. It wasn’t light and carefree; it was heavy and burdened. That’s when I saw my parents talking with my aunt. I couldn’t figure out why she was there, but they all looked very upset. We didn’t go on vacation that day. My grandmother had died. I hated my parents for inviting all of our family over to laugh and tell stories. Why was everyone so happy? It took a long time for me to realize that that was just how my family dealt with death.
 

How might the events of the passage be viewed differently if it were told by the narrator’s parents?

A
The story would be more suspenseful because readers wouldn’t know what exactly was going on.
B
The story would become ironic because the parents know more than the son does.
C
The main concern of the narrator would be how to tell their son about what happened to his grandmother.
D
The tone would be happy because the parents didn’t want to go on the vacation as much as the son did.
E
The parents would have less to think about because they have most likely dealt with death more than the son has.
Question 38 Explanation: 
Answer choice (C) is correct because it is the only answer that accurately portrays the real conflict of the story from a different perspective. Answer choice (A) is incorrect because the parents would arguably know more about what is going on than the son; this, however, wouldn’t necessarily make the passage ironic, as proposed in answer choice (B). Answer choice (D) is incorrect because readers have no reason to believe that the parents didn’t want to go on vacation; and answer choice (E) is incorrect because the parents have a lot to think about now that someone in their family has died.
Question 39

Use the graph below to answer questions 39–40.

The chart compares American recycling rates for select materials (paper, glass, metals and plastics) over a fifty-year time frame (1960–2010), using ten-year intervals.

Which of the following statements can be made with certainty about recycling habits prior to 1970 based only on the graph?

A
Recycling of some products increased in 1960 from the previous years.
B
There was no recycling during the years between 1960 and 1970.
C
There is no data indicated on the chart to show if 1960 held an increase or decrease in recycling for any of the materials.
D
There was no recycling before 1960.
E
The general population was not environmentally aware of recycling in the years prior to 1960.
Question 39 Explanation: 
Since we do not have data about what happened before 1960, we cannot tell if recycling increased or decreased in 1960. This eliminates choices (A), (D), and (E). Although the chart does not show the amount of recycling in the years between 1960 and 1970, it would be a mistake to assume that all recycling stopped during those years and then resumed in 1970; the study simply decided to portray the data for each tenth year.
Question 40

What is the primary conclusion that readers can draw from this graph?

A
People in this community have steadily increased their trash output over the duration of the study.
B
Recycling has become less of a priority in this community throughout the years covered in this study.
C
As time increases, so does the community’s passion for saving the environment.
D
Recycling has generally increased over time in the community where this data was taken.
E
Recycling was increasing for a while as time passed, but it has hit a plateau recently, showing little to know overall increase in the last few years.
Question 40 Explanation: 
Answer choice (D) is correct because the graph displays a positive correlation between time and percentage of trash recycled. Answer choice (C) might be tempting because it provides a plausible explanation for the data in the graph; however, the graph itself doesn’t remark on what has caused the increase over the years, which could simply be a product of more awareness and better recycling processes rather than increased care for the environment. The other answer choices are directly refuted by the data in the graph.
Question 41

Read the passage below; then answer questions 41–45.

The recent film “Hidden Figures” has brought a lot of much-earned recognition to one of America’s greatest mathematicians, Katherine Johnson. In a time when the country was still segregated, Katherine Johnson challenged racial and gender barriers to become one of the most trusted mathematicians at NASA during John Glenn’s 1961 Mercury Mission. As is famously portrayed in the 2017 film, NASA entrusted Johnson with the final calculations that brought John Glenn back to Earth safely and pinpointed where he would land for safe recovery. Katherine Johnson faced discrimination every day and still rose to be one of the most important behind-the-scenes figures in America’s burgeoning space program. The recent film has Americans wondering why stories like Johnson’s aren’t taught more specifically in school as everyone can benefit from such a story of resilience and strength amidst a fragile social climate. Perhaps stories like this challenge the white patriarchy in America and threaten to break the wheel of privilege, but that’s not a bad thing. Stories like Johnson’s would also help young women, and especially young women of color, connect with role models who have excelled in the fields of science and mathematics.
 

The primary purpose of this passage is to

A
lambast the American educational system that fails to inform children about unsung American heroes like Katherine Johnson.
B
celebrate an unsung hero who could be a great role model for young Americans.
C
argue that more women should study science and mathematics.
D
refute the argument that science and mathematics are fields better served for men.
E
criticize a film that took too many liberties with an already amazing story of perseverance and intelligence.
Question 41 Explanation: 
Answer choice (B) is correct because the passage glowingly discusses some of Katherine Johnson’s accomplishments while applauding heroes like her that could become role models for young women, especially young women of color. Answer choice (C) might be tempting because it’s pretty clear that the author would agree with that statement, but that is not the primary focus of the passage. Similarly, answer choice (D) expresses an opinion that the author would likely agree with, but the passage is not presented as a refutation of any counter-argument. The other answer choices represent ideas and/or opinions that are not supported by the passage.
Question 42
The recent film “Hidden Figures” has brought a lot of much-earned recognition to one of America’s greatest mathematicians, Katherine Johnson. In a time when the country was still segregated, Katherine Johnson challenged racial and gender barriers to become one of the most trusted mathematicians at NASA during John Glenn’s 1961 Mercury Mission. As is famously portrayed in the 2017 film, NASA entrusted Johnson with the final calculations that brought John Glenn back to Earth safely and pinpointed where he would land for safe recovery. Katherine Johnson faced discrimination every day and still rose to be one of the most important behind-the-scenes figures in America’s burgeoning space program. The recent film has Americans wondering why stories like Johnson’s aren’t taught more specifically in school as everyone can benefit from such a story of resilience and strength amidst a fragile social climate. Perhaps stories like this challenge the white patriarchy in America and threaten to break the wheel of privilege, but that’s not a bad thing. Stories like Johnson’s would also help young women, and especially young women of color, connect with role models who have excelled in the fields of science and mathematics.
 

Which of the following quotes from the passage represents an opinion rather than a fact?

A
“In a time when the country was still segregated, Katherine Johnson challenged racial and gender barriers to become one of the most trusted mathematicians at NASA during John Glenn’s 1961 Mercury Mission.”
B
“As is famously portrayed in the 2017 film, NASA entrusted Johnson with the final calculations that brought John Glenn back to Earth safely and pinpointed where he would land for safe recovery.”
C
“Katherine Johnson faced discrimination every day and still rose to be one of the most important behind-the-scenes figures in America’s burgeoning space program.”
D
“Perhaps stories like this challenge the white patriarchy in America and threaten to break the wheel of privilege, but that’s not a bad thing.”
E
“Stories like Johnson’s would also help young women, and especially young women of color, connect with role models who have excelled in the fields of science and mathematics.”
Question 42 Explanation: 
Answer choice (D) is correct because this is the only sentence when the author allows himself to editorialize and make a biased assumption about the American education system as well as a recommendation for how to improve it. While the entire passage has a tone of praise for Katherine Johnson, each of the other answer choices represents a factual statement made by the author.
Question 43
The recent film “Hidden Figures” has brought a lot of much-earned recognition to one of America’s greatest mathematicians, Katherine Johnson. In a time when the country was still segregated, Katherine Johnson challenged racial and gender barriers to become one of the most trusted mathematicians at NASA during John Glenn’s 1961 Mercury Mission. As is famously portrayed in the 2017 film, NASA entrusted Johnson with the final calculations that brought John Glenn back to Earth safely and pinpointed where he would land for safe recovery. Katherine Johnson faced discrimination every day and still rose to be one of the most important behind-the-scenes figures in America’s burgeoning space program. The recent film has Americans wondering why stories like Johnson’s aren’t taught more specifically in school as everyone can benefit from such a story of resilience and strength amidst a fragile social climate. Perhaps stories like this challenge the white patriarchy in America and threaten to break the wheel of privilege, but that’s not a bad thing. Stories like Johnson’s would also help young women, and especially young women of color, connect with role models who have excelled in the fields of science and mathematics.
 

Which of the following words could be swapped in for “burgeoning” in the passage while most nearly maintaining the original meaning of the sentence?

A
diminishing
B
beginning
C
incipient
D
maturing
E
expanding
Question 43 Explanation: 
In the context of the passage, “burgeoning” is meant to indicate that while America’s space program was not yet completely established, it was growing quickly. For this reason, answer choice (E), “expanding,” is the best synonym to use. Answer choices (B) and (C) are both synonyms of “burgeoning,” but they don’t address the rapid growth expressed in the passage. Answer choice (D), “maturing,” is also a synonym on “burgeoning,” but “maturing” carries the connotation that it was still immature, or not functioning properly, a sentiment that is not shared by the passage (unless that immaturity refers to segregation, but that would add a snarky tone to the passage that doesn’t exist anywhere else). Answer choice (A) doesn’t mean the same thing as “burgeoning,” so it’s incorrect.
Question 44
The recent film “Hidden Figures” has brought a lot of much-earned recognition to one of America’s greatest mathematicians, Katherine Johnson. In a time when the country was still segregated, Katherine Johnson challenged racial and gender barriers to become one of the most trusted mathematicians at NASA during John Glenn’s 1961 Mercury Mission. As is famously portrayed in the 2017 film, NASA entrusted Johnson with the final calculations that brought John Glenn back to Earth safely and pinpointed where he would land for safe recovery. Katherine Johnson faced discrimination every day and still rose to be one of the most important behind-the-scenes figures in America’s burgeoning space program. The recent film has Americans wondering why stories like Johnson’s aren’t taught more specifically in school as everyone can benefit from such a story of resilience and strength amidst a fragile social climate. Perhaps stories like this challenge the white patriarchy in America and threaten to break the wheel of privilege, but that’s not a bad thing. Stories like Johnson’s would also help young women, and especially young women of color, connect with role models who have excelled in the fields of science and mathematics.
 

The phrase “break the wheel” can most accurately be described as a(n)

A
metaphor
B
anecdote
C
example of irony
D
invective
E
cacophony
Question 44 Explanation: 
The phrase “break the wheel” is a metaphor (A) comparing the pattern of white male leadership in America to a wheel in motion. In the context of the passage, the author hopes that stories like Johnson’s will help “break the wheel,” and pave the way for women and people of color to run the country. Answer choice (B) is incorrect because the phrase is not a specific story that supports the author’s point, and the phrase is also not an example of irony, as suggested by answer choice (C). There is no invective (D) in the phrase because invective is intentionally harsh or vitriolic language, and there is also no cacophony (E), or repetition of harsh sounds, in the phrase.
Question 45
The recent film “Hidden Figures” has brought a lot of much-earned recognition to one of America’s greatest mathematicians, Katherine Johnson. In a time when the country was still segregated, Katherine Johnson challenged racial and gender barriers to become one of the most trusted mathematicians at NASA during John Glenn’s 1961 Mercury Mission. As is famously portrayed in the 2017 film, NASA entrusted Johnson with the final calculations that brought John Glenn back to Earth safely and pinpointed where he would land for safe recovery. Katherine Johnson faced discrimination every day and still rose to be one of the most important behind-the-scenes figures in America’s burgeoning space program. The recent film has Americans wondering why stories like Johnson’s aren’t taught more specifically in school as everyone can benefit from such a story of resilience and strength amidst a fragile social climate. Perhaps stories like this challenge the white patriarchy in America and threaten to break the wheel of privilege, but that’s not a bad thing. Stories like Johnson’s would also help young women, and especially young women of color, connect with role models who have excelled in the fields of science and mathematics.
 

Which of the following statements most effectively supports the conclusion made by the author in the passage?

A
While Katherine Johnson was a great mathematician, there were a lot of people who helped John Glenn successfully enter space and return safely.
B
"Hidden Figures” was nominated for an Oscar for Best Picture in 2017.
C
Science and mathematics are still fields dominated by men.
D
In 2015, Katherine Johnson was awarded the Presidential Medal of Freedom by President Barack Obama.
E
In the recent November 2018 elections, America voted in a more diverse group of new representatives and senators than any other time in the country’s history.
Question 45 Explanation: 
Since the conclusion at the end of the passage is that stories like Johnson’s would lead to more women (and women of color) entering the fields of science and mathematics, the fact that these fields are still dominated by (white) men is the most effective piece of supporting evidence. Answer choices (B), (D), and (E) are all relevant facts that could fit within the passage without much adjustment, but none of them directly support the author’s conclusion as well as answer choice (C). Answer choice (A) undermines Katherine Johnson’s contribution to America’s space program, which goes against the tone and overall point of the passage.
Question 46

Read the passages below; then answer questions 46–50.

Passage 1
On paper, it may seem that the goal of farming is to harvest as many crops from your land as you can. The logic is that the more you reap, the more you sow, and the more you thrive. There is a such thing as “over farming,” though, whereby a farmer overworks his or her land and makes it less yielding for the next year’s harvest. The most famous incident of farmers “overworking” their land in American history is the Dust Bowl of the 1930’s. In an attempt to combat the Great Depression, many farmers over-plowed their land, making the soil too fine to properly maintain its structure and fertility. This made it much easier for winds to blow through and demolish everything the farmers were working to accomplish. During the Dust Bowl, fine soil from farms was being lifted by the wind and blown miles away. It’s hard for your seeds to turn into crops if they’re flying through the air to another state.

Passage 2
Not everyone realizes that successful farmers need to create a harmonious relationship with their land. Squeezing your land for everything it has is a short-term solution that inhibits long-term successful farming. When soil is overworked, it becomes less fertile and capable of nurturing plant life, leading to a decrease in productivity every year and ultimately to unfarmable land. The best farmers understand that as much as they need to grow crops for this year’s harvest, they need to maintain their farm so they can have a lucrative harvest next year. This means taking care to not plant crops excessively, over-plow the land, or generally put an unnecessary strain on the soil. When farmers are able to balance their current harvest with their future potential for harvest, they can create more long-term, sustainable success for themselves and their families.
 

Which of the following statements best describes the relationship between the two passages?

A
Passage 1 presents a problem and its causes, while Passage 2 discusses the solutions to that problem.
B
Passage 1 is supported by objective data, while Passage 2 is supported by unsubstantiated opinion.
C
The two passages present two different opinions on the same topic.
D
Passage 1 is very biased, while Passage 2 remains objective and factual.
E
Passage 2 is very specific in its evidence whereas Passage 1 remains very general.
Question 46 Explanation: 
Answer choice (A) is correct because both passages address the same issue, with Passage 1 focusing on what causes farms to be overworked, while Passage 2 focuses on how farmers work to avoid overworking their land. Since the two passages represent the same general idea, answer choice (C) is incorrect. Similarly, answer choice (B), (D), and (E) are incorrect because the two passages contain a similar level of objectivism and specificity.
Question 47
Passage 1
On paper, it may seem that the goal of farming is to harvest as many crops from your land as you can. The logic is that the more you reap, the more you sow, and the more you thrive. There is a such thing as “over farming,” though, whereby a farmer overworks his or her land and makes it less yielding for the next year’s harvest. The most famous incident of farmers “overworking” their land in American history is the Dust Bowl of the 1930’s. In an attempt to combat the Great Depression, many farmers over-plowed their land, making the soil too fine to properly maintain its structure and fertility. This made it much easier for winds to blow through and demolish everything the farmers were working to accomplish. During the Dust Bowl, fine soil from farms was being lifted by the wind and blown miles away. It’s hard for your seeds to turn into crops if they’re flying through the air to another state.

Passage 2
Not everyone realizes that successful farmers need to create a harmonious relationship with their land. Squeezing your land for everything it has is a short-term solution that inhibits long-term successful farming. When soil is overworked, it becomes less fertile and capable of nurturing plant life, leading to a decrease in productivity every year and ultimately to unfarmable land. The best farmers understand that as much as they need to grow crops for this year’s harvest, they need to maintain their farm so they can have a lucrative harvest next year. This means taking care to not plant crops excessively, over-plow the land, or generally put an unnecessary strain on the soil. When farmers are able to balance their current harvest with their future potential for harvest, they can create more long-term, sustainable success for themselves and their families.
 

Which of the following best describes the organizational structure of Passage 1?

A
The passage presents a commonly held opinion, and then refutes it.
B
The passage starts with an anecdote, and then discusses how that anecdote supports the author’s opinion.
C
The passage provides a step-by-step method of avoiding a potential problem.
D
The passage presents an idea, then provides an example to support it.
E
The passage uses emotionally appeals to the audience with pathological appeals.
Question 47 Explanation: 
Answer choice (D) is correct because the passage presents the concept of overworking a farm, then describes a well-known historical event that was (at least in part) a result of this problem. The passage does not refute a popular opinion (A), use an anecdote (B), or provide a step-by-step process (C). The author’s reference to the Dust Bowl is also not intended to be pathological and emotional, but rather it is intended to provide a logical example of what can happen when a farm is overworked.
Question 48
Passage 1
On paper, it may seem that the goal of farming is to harvest as many crops from your land as you can. The logic is that the more you reap, the more you sow, and the more you thrive. There is a such thing as “over farming,” though, whereby a farmer overworks his or her land and makes it less yielding for the next year’s harvest. The most famous incident of farmers “overworking” their land in American history is the Dust Bowl of the 1930’s. In an attempt to combat the Great Depression, many farmers over-plowed their land, making the soil too fine to properly maintain its structure and fertility. This made it much easier for winds to blow through and demolish everything the farmers were working to accomplish. During the Dust Bowl, fine soil from farms was being lifted by the wind and blown miles away. It’s hard for your seeds to turn into crops if they’re flying through the air to another state.

Passage 2
Not everyone realizes that successful farmers need to create a harmonious relationship with their land. Squeezing your land for everything it has is a short-term solution that inhibits long-term successful farming. When soil is overworked, it becomes less fertile and capable of nurturing plant life, leading to a decrease in productivity every year and ultimately to unfarmable land. The best farmers understand that as much as they need to grow crops for this year’s harvest, they need to maintain their farm so they can have a lucrative harvest next year. This means taking care to not plant crops excessively, over-plow the land, or generally put an unnecessary strain on the soil. When farmers are able to balance their current harvest with their future potential for harvest, they can create more long-term, sustainable success for themselves and their families.
 

Based on the context of Passage 1, “yielding” most nearly means which of the following?

A
tenable
B
fertile
C
barren
D
sparse
E
lavish
Question 48 Explanation: 
Answer choice (B) is correct because, within the context of the passage, “yielding” refers to the fertility of the soil in future years. Answer choice (E) might be tempting, but “lavish” implies a surplus, which goes against the point that the author is trying to make. Answer choices (C) and (D) are antonyms to “yielding,” while answer choice (A), “tenable,” means defendable, which doesn’t exactly fit the original sentence.
Question 49
Passage 1
On paper, it may seem that the goal of farming is to harvest as many crops from your land as you can. The logic is that the more you reap, the more you sow, and the more you thrive. There is a such thing as “over farming,” though, whereby a farmer overworks his or her land and makes it less yielding for the next year’s harvest. The most famous incident of farmers “overworking” their land in American history is the Dust Bowl of the 1930’s. In an attempt to combat the Great Depression, many farmers over-plowed their land, making the soil too fine to properly maintain its structure and fertility. This made it much easier for winds to blow through and demolish everything the farmers were working to accomplish. During the Dust Bowl, fine soil from farms was being lifted by the wind and blown miles away. It’s hard for your seeds to turn into crops if they’re flying through the air to another state.

Passage 2
Not everyone realizes that successful farmers need to create a harmonious relationship with their land. Squeezing your land for everything it has is a short-term solution that inhibits long-term successful farming. When soil is overworked, it becomes less fertile and capable of nurturing plant life, leading to a decrease in productivity every year and ultimately to unfarmable land. The best farmers understand that as much as they need to grow crops for this year’s harvest, they need to maintain their farm so they can have a lucrative harvest next year. This means taking care to not plant crops excessively, over-plow the land, or generally put an unnecessary strain on the soil. When farmers are able to balance their current harvest with their future potential for harvest, they can create more long-term, sustainable success for themselves and their families.
 

Which of the following analogies best exemplifies the author’s main idea in Passage 2?

A
A bird in the hand is worth two in the bush.
B
Farmers need to be careful how much they work their soil.
C
Children need to be hugged to feel love, but squeezing them too hard could be harmful to them.
D
A broken clock is still right twice a day.
E
A flower needs both sunlight and darkness to properly grow.
Question 49 Explanation: 
Answer choice (C) is correct because it presents a comparable example to support the author’s main idea in Passage 2. Much like squeezing a child too hard could cause it harm, farming your land too much could cause it harm. Answer choice (B) might be tempting, but it’s not actually an anecdote; rather it is just a statement of the author’s main idea. Answer choice (E) might also be tempting because it mentions plants, but the analogy itself doesn’t make much sense (flowers do not require darkness to grow). Answer choices (A) and (D) are simply cliched phrases that don’t apply to Passage 2.
Question 50
Passage 1
On paper, it may seem that the goal of farming is to harvest as many crops from your land as you can. The logic is that the more you reap, the more you sow, and the more you thrive. There is a such thing as “over farming,” though, whereby a farmer overworks his or her land and makes it less yielding for the next year’s harvest. The most famous incident of farmers “overworking” their land in American history is the Dust Bowl of the 1930’s. In an attempt to combat the Great Depression, many farmers over-plowed their land, making the soil too fine to properly maintain its structure and fertility. This made it much easier for winds to blow through and demolish everything the farmers were working to accomplish. During the Dust Bowl, fine soil from farms was being lifted by the wind and blown miles away. It’s hard for your seeds to turn into crops if they’re flying through the air to another state.

Passage 2
Not everyone realizes that successful farmers need to create a harmonious relationship with their land. Squeezing your land for everything it has is a short-term solution that inhibits long-term successful farming. When soil is overworked, it becomes less fertile and capable of nurturing plant life, leading to a decrease in productivity every year and ultimately to unfarmable land. The best farmers understand that as much as they need to grow crops for this year’s harvest, they need to maintain their farm so they can have a lucrative harvest next year. This means taking care to not plant crops excessively, over-plow the land, or generally put an unnecessary strain on the soil. When farmers are able to balance their current harvest with their future potential for harvest, they can create more long-term, sustainable success for themselves and their families.
 

Which of the following best describes the tone of each passage?

A
Passage 1 is derisive, while Passage 2 is belligerent
B
Passage 1 is facetious, while Passage 2 is incensed
C
Passage 1 is joyful, while Passage 2 is jaded
D
Passage 1 is patronizing, while Passage 2 is reverent
E
Passage 1 is cautionary, while Passage 2 is encouraging
Question 50 Explanation: 
Answer choice (E) is the most accurate representation of each passage’s tone. The author of Passage 1 is providing a warning about the downfalls of overworking soil, and is therefore “cautionary.” Passage 2, conversely, is optimistic and provides “encouraging” advice for how farmers can create a better living for themselves.
Question 51

Read the passage below; then answer questions 51–54.

Many governments have chosen to respond to climate change by adopting a "wait-and-see" approach, but responsible governments are trying to respond to climate change by pursuing research programs to improve scientific knowledge and develop technological options, by regulating greenhouse gas emissions, or by engaging in a combination of research and regulation. The United States has invested in research and subsidized the development of carbon-removal and alternative energy technologies. Furthermore, some programs that were intended to achieve other goals, such as pollution reduction, energy independence, and the limitation of soil erosion, also discourage emissions or encourage the removal of greenhouse gases from the atmosphere.

Should a government decide to control emissions, it may choose from a broad menu of regulatory approaches. One option is direct controls, which set emissions standards for equipment and processes, require households and businesses to use specific types of equipment, or prohibit them from using others. A government could also adopt more indirect, incentive-based approaches, either singly or in combination—for example, by restricting overall quantities of emissions through a system of permits or by raising the price of emissions through fees or taxes. Incentive-based approaches are generally more cost-effective than direct controls as a means of regulating greenhouse gas emissions.

The main point that the author is trying to make in the passage is that

A
there are many ways for a government to address climate change, but they need to act.
B
there is no evidence that climate change in man-made or that it is negatively affecting the Earth.
C
the only real way to respond to climate change is through regulating greenhouse gas emissions.
D
it would be imprudent for governments to actively respond to climate change until more research is conducted.
E
the US government needs to work harder to combat climate change.
Question 51 Explanation: 
Answer choice (A) is correct because the passage offers several ways in which governments can respond to climate change, and only deems a common “wait-and-see approach” as imprudent. The passage doesn’t reflect on the validity of climate change (B), nor does it identify one method of action as the “only real way” (C). Answer choice (D) is directly refuted by the passage and answer choice (E) reflects a judgment that is not specifically made in the passage.
Question 52
Many governments have chosen to respond to climate change by adopting a "wait-and-see" approach, but responsible governments are trying to respond to climate change by pursuing research programs to improve scientific knowledge and develop technological options, by regulating greenhouse gas emissions, or by engaging in a combination of research and regulation. The United States has invested in research and subsidized the development of carbon-removal and alternative energy technologies. Furthermore, some programs that were intended to achieve other goals, such as pollution reduction, energy independence, and the limitation of soil erosion, also discourage emissions or encourage the removal of greenhouse gases from the atmosphere.

Should a government decide to control emissions, it may choose from a broad menu of regulatory approaches. One option is direct controls, which set emissions standards for equipment and processes, require households and businesses to use specific types of equipment, or prohibit them from using others. A government could also adopt more indirect, incentive-based approaches, either singly or in combination—for example, by restricting overall quantities of emissions through a system of permits or by raising the price of emissions through fees or taxes. Incentive-based approaches are generally more cost-effective than direct controls as a means of regulating greenhouse gas emissions.

In the context of the passage, what is the meaning of “incentive-based”?

A
giving people an example of what will happen if they don’t comply
B
persuading people to change using rewards
C
requiring everyone to follow the same rules
D
using research to decide the best course of action
E
threatening imprisonment if someone does not comply
Question 52 Explanation: 
After using the term “incentive-based,” the author discusses raising taxes on emissions to incentivize, or reward, people who emit less greenhouse gases. This means that choice (B) is correct. Choice (C) is incorrect because the government is not requiring people to follow a rule, they are simply taxing them if they don’t meet a standard. Choices (A), (D), and (E) are not supported by the passage.
Question 53
Many governments have chosen to respond to climate change by adopting a "wait-and-see" approach, but responsible governments are trying to respond to climate change by pursuing research programs to improve scientific knowledge and develop technological options, by regulating greenhouse gas emissions, or by engaging in a combination of research and regulation. The United States has invested in research and subsidized the development of carbon-removal and alternative energy technologies. Furthermore, some programs that were intended to achieve other goals, such as pollution reduction, energy independence, and the limitation of soil erosion, also discourage emissions or encourage the removal of greenhouse gases from the atmosphere.

Should a government decide to control emissions, it may choose from a broad menu of regulatory approaches. One option is direct controls, which set emissions standards for equipment and processes, require households and businesses to use specific types of equipment, or prohibit them from using others. A government could also adopt more indirect, incentive-based approaches, either singly or in combination—for example, by restricting overall quantities of emissions through a system of permits or by raising the price of emissions through fees or taxes. Incentive-based approaches are generally more cost-effective than direct controls as a means of regulating greenhouse gas emissions.
 

What foundational assumption is NOT made in the selection?

A
That climate change is actually taking place
B
That climate change is a significant problem
C
That government is the best entity to solve the problem of climate change
D
That greenhouse gases are a primary cause of climate change
E
That energy independence is vital to controlling climate change
Question 53 Explanation: 
While some debate whether or not climate change is actually taking place or that greenhouse gases are its cause, the author begins with the underlying premise that both are true. When he suggests solutions in the second paragraph, each of them involves the government, showing that he assumes that the government is the best entity to provide a solution to the situation. He mentions energy independence, but he does not indicate that it is vital to controlling the situation—only that it may indirectly help.
Question 54
Many governments have chosen to respond to climate change by adopting a "wait-and-see" approach, but responsible governments are trying to respond to climate change by pursuing research programs to improve scientific knowledge and develop technological options, by regulating greenhouse gas emissions, or by engaging in a combination of research and regulation. The United States has invested in research and subsidized the development of carbon-removal and alternative energy technologies. Furthermore, some programs that were intended to achieve other goals, such as pollution reduction, energy independence, and the limitation of soil erosion, also discourage emissions or encourage the removal of greenhouse gases from the atmosphere.

Should a government decide to control emissions, it may choose from a broad menu of regulatory approaches. One option is direct controls, which set emissions standards for equipment and processes, require households and businesses to use specific types of equipment, or prohibit them from using others. A government could also adopt more indirect, incentive-based approaches, either singly or in combination—for example, by restricting overall quantities of emissions through a system of permits or by raising the price of emissions through fees or taxes. Incentive-based approaches are generally more cost-effective than direct controls as a means of regulating greenhouse gas emissions.
 

The passage supports which of the following statements about climate change and potential responses to it?

select all that apply

A
The United States is leading the global charge to respond to climate change.
B
Climate change is real and needs to be addressed.
C
Climate change is controllable, but it requires immediate action.
D
Incentive-based approaches are generally most cost effective than other responses.
Question 54 Explanation: 
The entire premise of the passage relies on climate change being real, so answer choice (B) is correct. Answer choice (D) is correct because the author directly states it in the final sentence of the passage. While the passage does mention the US government, it doesn’t comment on whether they are “leading the global charge” against climate change, so answer choice (A) is incorrect. While the author does recommend “immediate action” in the passage, he never goes so far as to say climate change is “controllable,” which makes answer choice (C) incorrect.
Question 55

The selection below is a page from an index of book, showing most of the items in the section of the book about Sculpture. Use the index page to answer questions 55–56.

SCULPTURE, 15–299
  • Berlin. 200–203
  • Bologna. 217–230
    • S. DOMENICO. S. Petronio; An Angel.
  • Bruges. 204–216
    • NOTRE DAME. Madonna. Finished before August.
  • Florence. 15–89
    • ACADEMY. David. Life size model of reclining Male Figure.
    • BARGELLO. Bacchus. E. Bust of Brutus. Tondo, Relief: Madonna. Apollo.
    • BOBOLI GARDENS, GROTTO. Four unfinished Figures.
    • CASA BUONARROTI. Reliefs: Centaurs and Lapithæ. E. Madonna. E.
    • DUOMO, BEHIND HIGH ALTAR. Pietà. L.
    • S. LORENZO, NEW SACRISTY. Madonna; Tombs of Lorenzo dei Medici, Duke of Urbino, and Giuliano, Duke of Nemours.
  • London. 245–287
    • BURLINGTON HOUSE, DIPLOMA GALLERY. Tondo, Relief: Madonna.
    • VICTORIA AND ALBERT MUSEUM. Cupid.
    • BEIT COLLECTION. Young Athlete (bronze).
  • Milan. 90–92
    • PRINCE TRIVULZIO. Small Slave (bronze).
  • Paris. 231–245
    • ROOM OF RENAISSANCE SCULPTURE. Two Slaves.
  • Rome. 93–199
    • Unfinished p.93–112
      • PALAZZO RONDANINI. Pietà
    • Finished p. 113–199
      • ST. PETER’S. Pietà.
      • SAINT PIETRO IN VINCOLI. Moses, Rachel, and Leah.
      • MARIA SOPRA MINERVA. Christ with Cross.
 

What is the orderly arrangement used in the Sculpture section of the book?

A
chronological order (time)
B
spatial order (location)
C
order of importance
D
comparison and contrast
E
statements and reasons
Question 55 Explanation: 
The main subcategories in this Sculpture section of the book are names of cities, indicating the location of the various sculptures. Therefore, the arrangement is best described as spatial.
Question 56
SCULPTURE, 15–299
  • Berlin. 200–203
  • Bologna. 217–230
    • S. DOMENICO. S. Petronio; An Angel.
  • Bruges. 204–216
    • NOTRE DAME. Madonna. Finished before August.
  • Florence. 15–89
    • ACADEMY. David. Life size model of reclining Male Figure.
    • BARGELLO. Bacchus. E. Bust of Brutus. Tondo, Relief: Madonna. Apollo.
    • BOBOLI GARDENS, GROTTO. Four unfinished Figures.
    • CASA BUONARROTI. Reliefs: Centaurs and Lapithæ. E. Madonna. E.
    • DUOMO, BEHIND HIGH ALTAR. Pietà. L.
    • S. LORENZO, NEW SACRISTY. Madonna; Tombs of Lorenzo dei Medici, Duke of Urbino, and Giuliano, Duke of Nemours.
  • London. 245–287
    • BURLINGTON HOUSE, DIPLOMA GALLERY. Tondo, Relief: Madonna.
    • VICTORIA AND ALBERT MUSEUM. Cupid.
    • BEIT COLLECTION. Young Athlete (bronze).
  • Milan. 90–92
    • PRINCE TRIVULZIO. Small Slave (bronze).
  • Paris. 231–245
    • ROOM OF RENAISSANCE SCULPTURE. Two Slaves.
  • Rome. 93–199
    • Unfinished p.93–112
      • PALAZZO RONDANINI. Pietà
    • Finished p. 113–199
      • ST. PETER’S. Pietà.
      • SAINT PIETRO IN VINCOLI. Moses, Rachel, and Leah.
      • MARIA SOPRA MINERVA. Christ with Cross.
 

In which city are the unfinished sculptures that the book discusses?

A
London
B
Bologna
C
Milan
D
Berlin
E
Rome
Question 56 Explanation: 
The unfinished sculptures are discussed on pages 93 to 112 and are listed in the index as a subsection of the city of Rome.
Once you are finished, click the button below. Any items you have not completed will be marked incorrect. Get Results
There are 56 questions to complete.
List
Return
Shaded items are complete.
12345
678910
1112131415
1617181920
2122232425
2627282930
3132333435
3637383940
4142434445
4647484950
5152535455
56End
Return